You are on page 1of 6

Chapter 1

1.10 (a) Amount paid first four years = 900,000(0.12) = $108,000


(b) Final payment = 900,000 + 900,000(0.12) = $1,008,000
1.18 F = ?; A = $100,000 + $125,000?; i = 15%; n = 3

1.30 (a) Early-bird payment = 10,000 10,000(0.10) = $9000


(b) Equivalent future amount = 9000(1 + 0.10) = $9900
Savings = 10,000 9900 = $100

1.33 (a) F = 1,800,000(1 + 0.10) (1 + 0.10) = $2,178,000


(b) Interest = 2,178,000 1,800,000 = $378,000

1.35 4,600,000 = P(1 + 0.10)(1 + 0.10)


P = $3,801,653
1.40 WACC = 0.30(8%) + 0.70(13%) = 11.5%

Chapter 2
2.5 F = 50,000(F/P,6%,14)
= 50,000(2.2609)
= $113,045
2.13 P = (110,000* 0.3)(P/A,12%,4)
= (33,000)(3.0373)
= $100,231
2.19 F = (200 90)(F/A,10%,8)
= 110(11.4359)
= $1,257,949

2.29 (a) CF3 = 70 + 3(4) = $82 ($82,000)


(b) P = 74(P/A,10%,10) + 4(P/G,10%,10)
= 74(6.1446) + 4(22.8913)
= $546.266 ($546,266)
F = 546.266(F/P,10%,10)
= 521.687(2.5937)
= $1416.850 ($1,416,850)
2.30 601.17 = A + 30(A/G,10%,9)
601.17 = A + 30(3.3724)
A = $500

2.37 P = 30,000[10/(1 + 0.06)] = $283,019

2.43 87,360 = 24,000(F/A,i,3)


(F/A,i,3) = 3.6400
For n = 3 in F/A column, 3.6400 is in 20% interest table. Therefore, i = 20% per year.
2.45 600,000 = 80,000(F/A,15%,n)
(F/A,15%,n) = 7.50
Interpolate in the 15% interest table or use a spreadsheet function. By spreadsheet, n = 5.4
years.
2.49 350,000 = 15,000(P/A,4%,n) + 21,700(P/G,4%,n)
Solve by trial and error in 4% interest table between 5 and 6 years to determine
n 6 years

Chapter 3
3.5 P = 200(P/A,10%,3)(P/F,10%,1) + 90(P/A,10%,3)(P/F,10%,5)
= 200(2.4869)(0.9091) + 90(2.4869)(0.6209)
= $591.14
3.11 (a) 2,000,000 = 25,000(F/P,10%,20) + A(F/A,10%,20)
2,000,000 = 25,000(6.7275) + A(57.2750)
A = $31,983 per year
(b) Yes. In fact, they will exceed their goal by $459,188
3.18 A = -2500(A/P,10%,10) + (700 200)(P/A,10%,4)(A/P,10%,10)
+ (2000 300)(F/A,10%,6)(A/F,10%,10)
= -2500(0.16275) + 500(3.1699)(0.16275) + 1700(7.7156)(0.06275)
= $674.14 per year
3.26 Move all cash flows to year 8 and set equal to $500. Then solve for x.
-40(F/A,10%,4)(F/P,10%,4) - W(F/P,10%,3) - 40(F/A,10%,3) = -500
-40(4.6410)(1.4641) - W(1.3310) - 40(3.3100) = -500
W = $71.98
3.31 Q4 = 25(F/A,10%,6) + 25(P/F,10%,1) + 50(P/A/10%,3)(P/F,10%,1)
= 25(7.7156) + 25(0.9091) + 50(2.4869)(0.9091)
= $328.66

3.46 16,000 = [8000 + 8000(P/A,10%,4) G(P/G,10%,4)](P/F,10%,1)


16,000 = [8000 + 8000(3.1699) G(4.3781)](0.9091)
3.9801G = -16,000 + 30,327
G = $3600
This is a negative gradient series
3.50 P1 = 470(P/A,10%,6) 50(P/G,10%,6) + 470(P/F,10%,7)
= 470(4.3553) 50(9.6842) + 470(0.5132)
= $1803.99
F = 1803.99(F/P,10%,7)
= 1803.99(1.9487)
= $3515
3.62 A = 2,000,000(A/F,10%,5) = 2,000,000(0.16380)
= $327,600
Answer is (b)

Chapter 4
4.5 (a) Quarter (b) Semiannual (c) Month (d) Week (e) Continuous
4.6 (a) Nominal; (b) Nominal; (c) Effective; (d) Nominal; (e) Effective; (f) Effective
4.22 F = 260,000(F/P,3%,12)
= 260,000(1.4258)
= $370,708
4.26 In $1 million units,
28 = 12(F/P,3%,16) + x(F/P,3%,12)
28 = 12(1.6047) + x(1.4258)
1.4258x = 8.7436
x = $6.1324 ($6,132,400)

4.37 300 = A(P/A,1.5%,12) + [375 -10(12)](P/F,1.5%,12)


300 = A(10.9075) + [255](0.8364)
10.9075A = 86.72
A = $7.95 per month

4.44 A per quarter = 3(1000) = $3000


F = 3000(F/A,1.5%,20)
= 3000(23.1237)
= $69,371
4.45 Chemical cost = 11(30) = $3300 per month
A = 2(950)(A/P,1%,36) + 3300
= 2(950)(0.03321) + 3300
= $3363.10 per month

4.54 F = 300,000(F/P,1%,4)(F/P,1.25%,8)
= 300,000(1.0406)(1.1045)
= $344,803

4.57 (a) P = 100(P/A,10%,5) + 160(P/A,14%,3)(P/F,10%,5)


= 100(3.7908) + 160(2.3216)(0.6209)
= 100(3.7908) + 160(1.4415)
= $609.72
(b) 609.72 = A(3.7908) + A(1.4415)
A = 609.72/5.2323
= $116.53 per year

You might also like